Sequence problems - solution check

  • Thread starter Thread starter Government$
  • Start date Start date
  • Tags Tags
    Sequence
Click For Summary
SUMMARY

The discussion focuses on the sequence \( (a_n) \) where \( \lim_{n\rightarrow +\infty} n(a_n) = 0 \). It concludes that \( \lim_{n\rightarrow +\infty}(1 + \frac{1}{n} + a_n)^n = e \). Additionally, it establishes that for any real numbers \( p \) and \( l \) (with \( l \neq -1 \) and \( l \neq 0 \)), the sequence \( (b_n) = \frac{n^{p \cos(n\pi)}}{(1 + l + a_n)^n} \) is properly defined after some \( n_0 \). The analysis employs the Stolz–Cesàro theorem to validate the conditions for \( b_n \) being well-defined.

PREREQUISITES
  • Understanding of limits in calculus, specifically \( \lim_{n\rightarrow +\infty} n(a_n) \).
  • Familiarity with exponential limits, particularly \( \lim_{n\rightarrow +\infty}(1 + \frac{1}{n})^n \).
  • Knowledge of the Stolz–Cesàro theorem for sequences.
  • Basic understanding of sequences and their convergence properties.
NEXT STEPS
  • Study the Stolz–Cesàro theorem in detail to understand its applications in sequence convergence.
  • Explore the properties of exponential functions and their limits, particularly in the context of sequences.
  • Investigate the implications of the limit \( \lim_{n\rightarrow +\infty} n(a_n) = 0 \) on the behavior of sequences.
  • Learn about the behavior of oscillating sequences, particularly those involving \( \cos(n\pi) \).
USEFUL FOR

Mathematicians, students studying calculus or real analysis, and anyone interested in advanced sequence convergence and limit properties.

Government$
Messages
87
Reaction score
1

Homework Statement


Let $$(a_n)$$ be a sequence such that $$\lim_{n\rightarrow +\infty}n(a_n)=0$$.
1) What is
$$\lim_{n\rightarrow +\infty}(1 + {\frac{1}{n}} + (a_n))^n$$
2) For which value of p and l, after some n is $$(b_n)=\frac{n^{p \ cos(n\pi)}}{(1 + l + (a_n))^n}$$ properly defined. p and l are real numbers.

The Attempt at a Solution



1) Since $$\lim_{n\rightarrow +\infty}n(a_n)=0$$. I have that for some $$n_0$$, for every $$n>n_0$$ $$-1<n(a_n)$$ hence $$\lim_{n\rightarrow +\infty}\frac{n}{1 + n(a_n)} = \infty$$. Therefore i have that $$\lim_{n\rightarrow +\infty}(1 + {\frac{1}{n}} + (a_n))^n = \lim_{n\rightarrow +\infty}(1 + {\frac{1}{\frac{n}{1 + n(a_n)}}})^{\frac{n}{1 + n(a_n)}(1 + n(a_n))}= e^1=e$$

2) Considering that p is not diving anything here, and that p is exponent to n it seems to me that p can have any value. Now let $$l\neq -1 \neq 0$$. Since $$ (1 + l + (a_n)) = (1 + l + \frac{n(a_n)}{n})$$ and i have that $$ \lim_{n\rightarrow +\infty} \frac{n(a_n)}{n} = \lim_{n\rightarrow +\infty} \frac{(n+1)(a_{n+1}) - n(a_n)}{1} = \lim_{n\rightarrow +\infty} ((n+1)(a_{n+1}) - n(a_n))= 0 - 0 = 0 $$ by Stolz–Cesàro theorem. This means that I have for some $$n_0$$, for every $$n>n_0$$ So have that for $$1 + l > 0$$, $$|{\frac{n(a_n)}{n}}| < 1+l $$ and for $$1 + l < 0$$ $$|{\frac{n(a_n)}{n}}| < -(1+l) $$. Therefore $$(1 + l + \frac{n(a_n)}{n}) \neq 0$$ for $$n>n_0$$ so i won't have any divison by 0. For $$l=0$$ i also get $$(1 + \frac{n(a_n)}{n}) \neq 0$$ for $$n>n_0$$ since $$ \lim_{n\rightarrow +\infty} \frac{n(a_n)}{n} = 0$$. For $$l=-1$$ i get $$(b_n)=\frac{n^{n}n^{p \ cos(n\pi)}}{(n(a_n))^n}$$ Since $$\lim_{n\rightarrow +\infty}n(a_n)=0$$ i have no division by 0.
To conclude fore every p,l from R, after some $$n_0$$ for all $$n>n_0$$ $$(b_n)$$ is properly defined.

Sorry for any typos.
 
Physics news on Phys.org
Government$ said:

Homework Statement


Let $$(a_n)$$ be a sequence such that $$\lim_{n\rightarrow +\infty}n(a_n)=0$$.
1) What is
$$\lim_{n\rightarrow +\infty}(1 + {\frac{1}{n}} + (a_n))^n$$
2) For which value of p and l, after some n is $$(b_n)=\frac{n^{p \ cos(n\pi)}}{(1 + l + (a_n))^n}$$ properly defined. p and l are real numbers.

The Attempt at a Solution



1) Since $$\lim_{n\rightarrow +\infty}n(a_n)=0$$. I have that for some $$n_0$$, for every $$n>n_0$$ $$-1<n(a_n)$$ hence $$\lim_{n\rightarrow +\infty}\frac{n}{1 + n(a_n)} = \infty$$. Therefore i have that $$\lim_{n\rightarrow +\infty}(1 + {\frac{1}{n}} + (a_n))^n = \lim_{n\rightarrow +\infty}(1 + {\frac{1}{\frac{n}{1 + n(a_n)}}})^{\frac{n}{1 + n(a_n)}(1 + n(a_n))}= e^1=e$$

2) Considering that p is not diving anything here, and that p is exponent to n it seems to me that p can have any value. Now let $$l\neq -1 \neq 0$$. Since $$ (1 + l + (a_n)) = (1 + l + \frac{n(a_n)}{n})$$ and i have that $$ \lim_{n\rightarrow +\infty} \frac{n(a_n)}{n} = \lim_{n\rightarrow +\infty} \frac{(n+1)(a_{n+1}) - n(a_n)}{1} = \lim_{n\rightarrow +\infty} ((n+1)(a_{n+1}) - n(a_n))= 0 - 0 = 0 $$ by Stolz–Cesàro theorem. This means that I have for some $$n_0$$, for every $$n>n_0$$ So have that for $$1 + l > 0$$, $$|{\frac{n(a_n)}{n}}| < 1+l $$ and for $$1 + l < 0$$ $$|{\frac{n(a_n)}{n}}| < -(1+l) $$. Therefore $$(1 + l + \frac{n(a_n)}{n}) \neq 0$$ for $$n>n_0$$ so i won't have any divison by 0. For $$l=0$$ i also get $$(1 + \frac{n(a_n)}{n}) \neq 0$$ for $$n>n_0$$ since $$ \lim_{n\rightarrow +\infty} \frac{n(a_n)}{n} = 0$$. For $$l=-1$$ i get $$(b_n)=\frac{n^{n}n^{p \ cos(n\pi)}}{(n(a_n))^n}$$ Since $$\lim_{n\rightarrow +\infty}n(a_n)=0$$ i have no division by 0.
To conclude fore every p,l from R, after some $$n_0$$ for all $$n>n_0$$ $$(b_n)$$ is properly defined.

Sorry for any typos.

Your answer for part 1) is correct, namely this portion of it:

$$\lim_{n\rightarrow +\infty}(1 + {\frac{1}{n}} + (a_n))^n = \lim_{n\rightarrow +\infty}(1 + {\frac{1}{\frac{n}{1 + n(a_n)}}})^{\frac{n}{1 + n(a_n)}(1 + n(a_n))}= e^1=e$$

Though I'm not sure I agree entirely with the argument you used to arrive at this.
 
Zondrina said:
Though I'm not sure I agree entirely with the argument you used to arrive at this.

You mean the way i proved $$\lim_{n\rightarrow +\infty}\frac{n}{1 + n(a_n)} = \infty$$H
How about this:
$$\lim_{n\rightarrow +\infty}\frac{n}{1 + n(a_n)} = \lim_{n\rightarrow +\infty} n * \lim_{n\rightarrow +\infty} \frac{1}{1 + n(a_n)}=\infty*1=\infty$$. I have $$\lim_{n\rightarrow +\infty}\frac{n}{1 + n(a_n)} = \lim_{n\rightarrow +\infty} n * \frac{1}{1 + n(a_n)} = \lim_{n\rightarrow +\infty} n * \lim_{n\rightarrow +\infty} \frac{1}{1 + n(a_n)}$$ by limit properties.
 
Last edited:
Question: A clock's minute hand has length 4 and its hour hand has length 3. What is the distance between the tips at the moment when it is increasing most rapidly?(Putnam Exam Question) Answer: Making assumption that both the hands moves at constant angular velocities, the answer is ## \sqrt{7} .## But don't you think this assumption is somewhat doubtful and wrong?

Similar threads

  • · Replies 8 ·
Replies
8
Views
2K
  • · Replies 2 ·
Replies
2
Views
2K
  • · Replies 2 ·
Replies
2
Views
2K
  • · Replies 7 ·
Replies
7
Views
2K
  • · Replies 1 ·
Replies
1
Views
2K
  • · Replies 7 ·
Replies
7
Views
2K
  • · Replies 5 ·
Replies
5
Views
1K
Replies
8
Views
3K
Replies
14
Views
2K
  • · Replies 34 ·
2
Replies
34
Views
3K